Разное

Задачи по физике решебник: Все решения к «Сборнику задач по общему курсу физики» В.С. Волькенштейн

Содержание

Электронная библиотека решебников






 

Добро пожаловать на , на сайт,
который представляет из себя большую и полностью бесплатную базу решений к задачам по курсу общей физики,теоретической механике,по математике.

Мы стараемся постоянно добавлять новый материал в надежде, что вы найдете у нас нужное решение, а также закончили занимаематься переводом решебника к «Сборнику задач по общему курсу физики И.Е. Иродова»,«Сборник задач по теоретической механике Мещерский И. В.»,«Сборник задач по теоретической механике Яблонский А.А.»,«Сборник задач по математике Филиппов А.Ф.» решений, которые вы не найдете на других сайтах. В будущем постараемся реализовать множество идей, которые станут хорошим дополнением к нашей базе.

По любым предложениям и замечаниям обращайтесь в нашу группу, по адресу vk.com/tasksall, или пишите на почту tasksall@mail. ru. Это очень поможет нам в дальнейшей работе над проектом.


 




Решебник к сборнику задач по физике А.Г. Чертов и А.А. Воробьев. 2001 год
Добавлено: 1 ноября 2013 г. в 15:00:15

Задачник по физике

Чертов А.Г., Воробьев А.А.

Задачник составлен в соответствии с действующей программой по курсу физики для втузов. В каждый раздел включено достаточное количество задач, трудность которых возрастает с увеличением порядкового номера. В начале каждого параграфа приводятся основные законы и формулы, даются примеры решения типовых задач. В конце задачника приведены ответы ко всем задачам.
В 7-е издание (6-е — 1997 г.) включено более 150 новых задач и примеров, существенно расширена теоретическая часть каждого раздела, а также внесены необходимые исправления.
Для студентов высших технических учебных заведений.

К списку задач »

1001
задача по физике с решениями

Добавлено: 13 июня 2013 г. в 10:12:47

 

1001 задача по физике с решениями

Гельфгат И.М., Генденштейн П.Э., Кирик П.А.

Книга содержит задачи по всему курсу элементарной физики. Они
градуированы следующим образом: задачи средней трудности, повышенной
трудности и олимпиадные задачи. К большинству задач приведены подробные
решения обучающего характера. Для итогового самоконтроля предложены тесты.
Книга предназначена для учащихся средних школ, лицеев и гимназий (в том
числе физико-математического профиля), студентов педагогических вузов,
преподавателей, а также лиц, занимающихся самообразованием.


К списку задач »

Решебник к сборнику задач по общей физике. И.Е. Иродов. 1979 год

Добавлено: 14 апреля 2013 г. в 22:15:07

 

Сборник задач по общему курсу физики

И.Е. Иродов.

Содержит свыше 2000 задач по всем разделам курса общей физики. Каждой теме предшествуют краткие теоретические сведения, в конце сборника приведены справочные таблицы. В новом издании (2-е — 1988 г.) материал сборника перекомпонован: механика, электромагнетизм, колебания и волны, оптика, квантовая физика и физика макросистем — в соответствии с современной концепцией изучения курса. Отдельные разделы сборника значительно переработаны, включен рад новых оригинальных задач, устранены замеченные неточности.Для студентов физических и инженерно-технических специальностей вузов, может быть использован во втузах с обычной программой по физике.

К списку задач »





Задачи по физике и математике с решениями и ответами

Задача по физикe — 1

Закрытый сосуд заполнен водой.{2}$ тоже является суммой квадратов трех натуральных чисел.


Подробнее

Задача по математике — 8

Существует ли конечное слово из букв русского алфавита, в котором нет двух соседних одинаковых подслов,

но таковые появляются при приписывании (как справа, так и слева) любой буквы русского алфавита.

Комментарий. Словом мы называем любую последовательность букв русского алфавита, не обязательно осмысленную, подсловом называется любой фрагмент слова. Например, АБВШГАБ — слово, а АБВ,Ш, ШГАБ — его подслова.


Подробнее

Задача по математике — 10

Кооператив получает яблочный и виноградный сок в одинаковых бидонах и выпускает яблочно-виноградный напиток в одинаковых банках. Одного бидона яблочного сока хватает ровно на 6 банок напитка, а одного бидона виноградного—ровно на 10. Когда рецептуру напитка изменили, одного бидона яблочного сока стало хватать ровно на 5 банок напитка. На сколько банок напитка хватит

теперь одного бидона виноградного сока? (Напиток водой не разбавляется.)


Подробнее

Физика. Вопросы — ответы. Задачи

Представлены первые три части выпускаемой серии методических
рекомендаций к решению задач по физике. Пособие призвано помочь
усвоить основные законы механики. Контрольные вопросы в
сочетании с приведенными ответами являются дополнением к основному
учебнику физики. Приведены алгоритмы решения большого количества
типовых задач. Кроме того, для самостоятельной работы подобраны
задачи различной трудности, которые могут быть использованы
учителем для работы с учениками.

Пособие предназначено учащимся 9-го класса для самостоятельной
работы, для подготовки к единому государственному экзамену,
а также может быть использовано абитуриентами для
подготовки к вступительным экзаменам.












Автор

Издательство


ООО «Физматлит»

Дата издания




2003

Кол-во страниц




352

Номер тома




1

Название тома




Механика

ISBN




978-5-9221-0316-9

Тематика




Физика. Химия. Биология (егэ,вуз)

Вес книги




360 г

№ в каталоге




316


Категории:
Для подготовки к ЕГЭ и поступлению в ВУЗ

Секреты решения задач ЕГЭ по физике

Вариант ЕГЭ по физике состоит из двух частей и включает в себя 32 задания.

В части 1 содержится 24 задания с кратким ответом, в которых ответ записывается в виде числа, двух чисел или слова, а также задания на установление соответствия и множественный выбор, в которых ответы необходимо записать в виде последовательности цифр.

Часть 2 содержит 8 заданий. Из них два задания с кратким ответом (25 и 26) и шесть заданий (27–32), для которых необходимо привести развернутый и обоснованный ответ.

В первой части – не только формулы и графики. Есть и необычные задания.

В задании 22 вы увидите фотографии или рисунки измерительных приборов. Чтобы сделать это задание, нужно уметь записывать показания приборов при измерении физических величин с учётом абсолютной погрешности измерений.

Задание 23 проверяет умение выбирать оборудование для проведения опыта по заданной гипотезе.

Завершает первую часть задание по астрономии на выбор нескольких утверждений из пяти предложенных.

Вторая часть работы посвящена решению задач: семи расчётных и одной качественной задачи.

Они распределяются по разделам следующим образом: 2 задачи по механике, 2 задачи по молекулярной физике и термодинамике, 3 задачи по электродинамике, 1 задача по квантовой физике.

Задания 25 и 26 – это расчётные задачи с кратким ответом. Задание 25 по молекулярной физике или электродинамике, а задача 26 – по квантовой физике.

Далее идут задания с развёрнутым ответом. Задание 27 – качественная задача, в которой решение представляет собой объяснение какого-либо факта или явления, основанное на физических законах и закономерностях. Качественная задача может быть по любому из разделов курса физики.

Следующие задачи строго распределены по определенным разделам физики.

Задание 28 – по механике или по молекулярной физике,

задание 29 –  по механике,

задание 30 – по МКТ и термодинамике,

задание 31 – по электродинамике,

задание 32 – преимущественно по оптике.

Для расчётных задач высокого уровня сложности (29–32) требуется анализ всех этапов решения. Здесь необходимо пользоваться большим числом законов и формул, вводить дополнительные обоснования в процессе решения. Способ решения задачи надо выбрать самостоятельно.

На нашем сайте размещены статьи по каждой задаче ЕГЭ. В них приведены не только типовые задания ЕГЭ по физике, но и показан подробный ход рассуждений, приводящих к решению задач. Каждое задание сопровождается ссылкой на необходимую теорию.

Рассказано о секретах решения каждой задачи ЕГЭ по физике.

Задание 1  Кинематика. Равномерное прямолинейное движение, равноускоренное прямолинейное движение, движение по окружности.

Задание 2 Силы в природе, законы Ньютона. Закон всемирного тяготения, закон Гука, сила трения

Задание 3  Закон сохранения импульса, кинетическая и потенциальные энергии, работа и мощность силы, закон сохранения механической энергии

Задание 4 Механическое равновесие, механические колебания и волны. Условие равновесия твёрдого тела, закон Паскаля, сила Архимеда,

Задание 5 Механика. Объяснение явлений; интерпретация результатов опытов, представленных в виде таблицы или графиков

Задание 6 Механика. Изменение физических величин в процессах. 

Задание 7  Механика. Установление соответствия между графиками и физическими величинами, между физическими величинами и формулами.

Задание 8 Основы термодинамики. Тепловое равновесие. Уравнение Клапейрона-Менделеева. Изопроцессы.

Задание 9  Термодинамика. Работа в термодинамике, первый закон термодинамики, КПД тепловой машины

Задание 10  Термодинамика, тепловое равновесие. Относительная влажность воздуха, количество теплоты

Задание 11  Термодинамика и молекулярно-кинетическая теория. Объяснение явлений; интерпретация результатов опытов, представленных в виде таблицы или графиков.

Задание 12  Термодинамика и молекулярно-кинетическая теория. Изменение физических величин в процессах; установление соответствия между графиками и физическими величинами, между физическими величинами и формулами. 

Задание 13 Электрическое поле, магнитное поле. Принцип суперпозиции электрических полей, магнитное поле проводника с током, сила Ампера, сила Лоренца, правило Ленца

Задание 14  Электричество. Закон сохранения электрического заряда, закон Кулона, конденсатор, сила тока, закон Ома для участка цепи, последовательное и параллельное соединение проводников, работа и мощность тока, закон Джоуля – Ленца

Задание 15  Электричество, магнетизм и оптика. Поток вектора магнитной индукции, закон электромагнитной индукции Фарадея, индуктивность, энергия магнитного поля катушки с током, колебательный контур, законы отражения и преломления света, ход лучей в линзе

Задание 16 Электродинамика. Объяснение явлений; интерпретация результатов опытов, представленных в виде таблицы или графиков

Задание 17 Электродинамика и оптика. Изменение физических величин в процессах

Задание 18  Электродинамика, оптика, специальная теория относительности. Установление соответствия между графиками и физическими величинами, между физическими величинами и формулами

Задание 19 Ядерная физика. Планетарная модель атома. Нуклонная модель ядра. Ядерные реакции.

Задание 20 Линейчатые спектры, фотоны, закон радиоактивного распада.

Задание 21 Квантовая физика. Изменение физических величин в процессах. Установление соответствия между графиками и физическими величинами, между физическими величинами и формулами

Задание 22 Механика — квантовая физика, методы научного познания

Задание 23 Механика — квантовая физика, методы научного познания

Задание 24 Элементы астрофизики. Солнечная система, звёзды, галактики

Задание 25 Молекулярная физика, термодинамика, электродинамика. Расчётная задача

Задание 26 Электродинамика, квантовая физика. Расчётная задача

Задание 27 Механика — квантовая физика. Качественная задача

Задание 28 Механика — квантовая физика. Расчётная задача

Задание 29 Механика. Расчетная задача

Задание 30 Молекулярная физика. Расчетная задача

Задание 31 Электродинамика. Расчетная задача

Задание 32 Электродинамика. Квантовая физика. Расчетная задача

 

 

 

Решебники/гдз для задачника по физике Л.А. Кирик 7,8,9,10,11 класс онлайн:•

ПОСМОТРЕТЬ ОНЛАЙН задачник кирик л.а 7 класс можно <ТУТ>

ПОСМОТРЕТЬ РЕШЕБНИК ОНЛАЙН для задачника кирик л.а за 7 класс можно <ТУТ> 

~~~~~~~~~~~~~~~~~~~~~~~~~~~~~~~~~~~~~~~~~~~~~~~~~~~~~~

ЗАДАЧНИК Л.А.КИРИК ЗА 8 КЛАСС ОНЛАЙН

СКАЧАТЬ задачник кирик л.а 8 класс в  pdf /jpeg/jpg можно <ТУТ>

ПОСМОТРЕТЬ ОНЛАЙН задачник кирик л.а 8 класс можно <ТУТ>

ПОСМОТРЕТЬ РЕШЕБНИК ОНЛАЙН для задачника кирик л.а за 8 класс можно <ТУТ> 

~~~~~~~~~~~~~~~~~~~~~~~~~~~~~~~~~~~~~~~~~~~~~~~~~~~~~~

ЗАДАЧНИК Л.А.КИРИК ЗА 9 КЛАСС ОНЛАЙН

СКАЧАТЬ задачник кирик л.а 9 класс в  pdf /jpeg/jpg можно <ТУТ>

ПОСМОТРЕТЬ ОНЛАЙН задачник кирик л.а 9 класс можно <ТУТ>

ПОСМОТРЕТЬ РЕШЕБНИК ОНЛАЙН для задачника кирик л.а за 9 класс можно <ТУТ> 

~~~~~~~~~~~~~~~~~~~~~~~~~~~~~~~~~~~~~~~~~~~~~~~~~~~~~~

ЗАДАЧНИК Л.А.КИРИК ЗА 10 КЛАСС ОНЛАЙН

СКАЧАТЬ задачник кирик л.а 10 класс в  pdf /jpeg/jpg можно <ТУТ>

ПОСМОТРЕТЬ ОНЛАЙН задачник кирик л.а 10 класс можно <ТУТ>

ПОСМОТРЕТЬ РЕШЕБНИК ОНЛАЙН для задачника кирик л.а за 10 класс можно <ТУТ> 

~~~~~~~~~~~~~~~~~~~~~~~~~~~~~~~~~~~~~~~~~~~~~~~~~~~~~~

ЗАДАЧНИК Л.А.КИРИК ЗА 11 КЛАСС ОНЛАЙН

СКАЧАТЬ задачник кирик л.а 11 класс в  pdf /jpeg/jpg можно <ТУТ>

ПОСМОТРЕТЬ ОНЛАЙН задачник кирик л.а 11 класс можно <ТУТ>

ПОСМОТРЕТЬ РЕШЕБНИК ОНЛАЙН для задачника кирик л.а за 11 класс можно <ТУТ> 

~~~~~~~~~~~~~~~~~~~~~~~~~~~~~~~~~~~~~~~~~~~~~~~~~~~~~~

Немного о задачниках:

Физика. Разноуровневые самостоятельные и контрольные работы для 7,8,9,10,11 класс.

Автор: Л. А. Кирик /Языки: Русский/Издательство: Илекса

От производителя:

 Книги содержат самостоятельные и контрольные работы по всем важнейшим темам курса физики 7,8,9,10,11 класса и предназначена для текущего контроля знаний учащихся. 

Работы состоят из нескольких вариантов четырех уровней сложности (начальный уровень, средний уровень, достаточный уровень и высокий уровень).

Как решать типовые задачи по физике

Обучаясь в школе, каждый сталкивается с решением задач по физике. Не всем дисциплина дается легко.

Бытует мнение, что для успешного решения задач по этому предмету, нужно досконально разбираться в физических процессах.3\). Также не забывайте про постоянные величины, например, ускорение свободного падения. В задачах на свободное падение о нем может быть не сказано ни слова, но оно предполагается в условиях и необходимо, чтобы их решить. Подумайте об этом, когда записываете все известные данные.

  • С столбце «СИ» приведите все данные в задаче к международным единицам измерения. Так как в международной системе основными единицами измерения массы считаются килограммы (кг), массу из приведенной выше задачи необходимо привести в нужное значение: 1 000 тонн = 1 000 000 кг. 
  • Нарисуйте схематичный рисунок. Он нужен не для всех задач. Но в тех, где упоминаются действующие на тело силы и векторы скоростей, изображение может существенно облегчить понимание процесса и натолкнуть на правильное решение.
  • Определите неизвестную величину, ту, что необходимо узнать, решив задание. Написав в столбике все, что известно в задаче, проведите черту под известными данными и пропишите ту величину, которую будете искать.
  • Подберите формулы. Это самый важный пункт в нашем алгоритме! Решение задачи после выбора формулы будет заключаться в математических вычислениях, которые имеют к физике лишь опосредованное отношение. На черновике выпишите те формулы, которые могут подойти для конкретной задачи и выберите ту, которая будет способствовать решению.
  • Математические вычисления. Остальное решение задачи сводится к математике. Нужно сделать необходимые преобразования и сокращения, если они нужны. Затем составить уравнение или систему уравнений. Остается только их решить и найти все неизвестные, а в конце искомую величину. Ответ обведите в прямоугольник. 


  • Источник: evrophiz.wordpress.com

    Примеры решения типовых задач по разделам

    Рассмотрим подробнее решение задач из разных разделов физики по предложенному алгоритму. И дадим все необходимые объяснения к каждой из них.

    Система абсолютно универсальна и подходит для решения заданий по динамике, кинематике, статике и другим разделам физики.

    Кинематика



    Источник: znanio.ru

    Кинематика — это раздел механики, который изучает математическое описание движения тел. 

    Данный раздел охватывает следующие темы:

    • равномерное и равноускоренное движение тел;
    • движение тела по окружности;
    • относительность движения;
    • свободное падение тел.

    Рассмотрим типовые задачи на каждую из этих тем.

    Равномерное и равноускоренное движение тел

    Для решения задач по этой теме нужно знать уравнение движения тела, понимать, что такое средняя, постоянная скорости и ускорение, уметь выяснять их векторное направление в конкретной задаче.

    Как правило, в задачах на равномерное и равноускоренное движение необходимо найти или пройденный путь (S), или скорость движения (V), или время (t).

    Задача:

    Поезд длиной 240 метров, двигаясь равномерно, прошел мост за 2 минуты. Какова была скорость поезда, если длина моста равна 360 метрам?

    Решение:

    1. Записываем известные нам данные:\( l_1=240\) м., \(l_2=360\) м., \(t=2\) мин., \(V\)=?
    2. Проводим необходимые преобразования времени до принятых в мире единиц измерения — секунд: 2 минуты = 120 секунд.
    3. Мы знаем, что скорость равномерного движения определяется по формуле: \(V=\frac st\) 
    4. Время нам известно, для того, чтобы найти скорость, нужно сначала определить путь пройденный поездом. Если мы схематично изобразим перемещение поезда по мосту, то увидим, что путь, пройденный поездом, равен длине самого поезда плюс длине самого моста, т.е. \(s=l_1+l_2\).
    5. Переходим к математическим вычислениям: \(s=240+360=600\) метров.
    6. \(V=600/120= 5\) м/с.

    Задача:

    При равноускоренном движении с начальной скоростью 5 м/с тело за 3 секунды прошло 20 метров. С каким ускорением двигалось тело? Какова его скорость в конце третьей секунды?

    Решение:

    1. Фиксируем данные известных нам величин: \(V_1=5 \) м/с, \(t=3\) с, s=20 м.2.\)
    2. Нам известна формула для определения скорости при равноускоренном движении: \(V_2=V_1+a\times t\)
    3. Все данные у нас для вычисления скорости есть, подставляем их в формулу и получаем скорость, равную \(8,3\) м/с.
    Движение тела по окружности

    Чтобы успешно решать задачи по этой теме, необходимо знать формулы, характеризующие движение тел по окружности. В задачах на движение тела по окружности обычно необходимо вычислить скорость, центростремительное ускорение, радиус или длину окружности.

    Задача:

    Каково центростремительное ускорение поезда, который движется по закругленной железной дороге радиусом 800 метров со скоростью 72 км/ч?

    Решение:

    1. Записываем вводные данные: \(R=800 м\), \(V=72\) км/ч, \(a\)=?
    2. Переводим скорость из км/ч в м/с, получаем 20 м/с.
    3. Мы знаем формулу, по которой можно определить центростремительное ускорение: \(a=\frac{V^2}R\)
    4. Все данные нам известны, подставляем числовые значения в формулу и получаем искомую нами величину, равную \(0,5 м/с^2\)
    Свободное падение тел

    Для решения задач по этой теме нужно знать закон движения при свободном падении и закономерность изменения скорости тела со временем, а также помнить про постоянную величину — коэффициент силы тяжести.

    В задачах на свободное падение тел может быть предложено найти скорость движения тела, высоту, с которой оно падало или время его движения.

    Задача:

    Камень брошен вниз с высоты \(85\) метров. Он летит со скоростью \(8\) м/с. С какой скоростью он ударяется о землю?

    Решение:

    1. Определяем известные и неизвестные нам данные: \(h=85\) метров, \(V_1=8\) м/с., \(V_2=?\) Мы помним, что на любое падающее тело воздействует коэффициент силы тяжести, равный \(9,8\) Н/кг.
    2. У нас есть все вводные для определения конечной скорости по формуле: \(V_2=V_1+g\times t\)
    3. Подставляем числовые значения в уравнение и получаем скорость тела в момент удара о землю, равную \(41,3\) м/с.
    Относительность движения

    Задачи на относительность движения всегда требует выбрать неподвижную систему координат, относительно которой и будут производиться все расчеты. В таких заданиях ученикам обычно предлагают найти относительную скорость объекта, минимальное время, продолжительность пути или длину объекта.

    Задача:

    Два поезда движутся навстречу друг другу по параллельным ж/д путям. Один — со скоростью 72 км/ч, другой — со скоростью 54 км/ч. Пассажир первого поезда отмечает, что второй проходит мимо него в течение 10 секунд. Определите длину второго поезда.

    Решение:

    1. Записываем известные нам данные: \(V_1=72\) км/ч, \(V_2=54\) км/ч, \(t=10\) с, \(l_2=?\)
    2. Переводим км/ч в м/с: \(V_1= 20\) м/с, \(V_2=15\) м/с.
    3. Определяем систему координат, от которой будем отталкиваться при вычислении искомой величины. Логично будет, если такой системой станет линейная система координат, связанная с первым поездом и направленная по ходу его движения. Получается, что второй поезд двигается со скоростью \(V_2=15\) м/с в направлении со скоростью \(V_1=20\) м/с.
    4. Находим общую скорость движения по формуле: \(V=V_1+V_2\) 
    5. Она равна \(35\) м/с.
    6. Определяем длину поезда по формуле: \(l_2=V\times t\)
    7. Получаем длину поезда, равную \(350\) метрам.2}\) вычисляем силу притяжения между книгами.
    8. Произведя математические вычисления получаем ответ: книги притягиваются друг к другу с силой приблизительно равной \(2,4\) Н.
    Сила упругости

    Задача: 

    К покоящейся на горизонтальной поверхности системе, которая состоит из куба массой 1 кг и 2-х пружин, приложена постоянная горизонтальная сила величиной 25 Ньютонов. Между кубом и поверхностью трения нет. Жесткость первой пружины составляет  \(450 Н/м\), жесткость второй пружины \(550 Н/м\). Определите удлинение пружин.

    Решение:

    1. Записываем в столбце «Дано» данные, которые нам известны: \(m=1\) кг,\( F=25\) Н, \(k_1=450\) Н/м, \(k_2=550\) Н/м, \(\Delta l_1=?\), \(\Delta l_2=? \) 
    2. Согласно 3-му закону Ньютона \(F=F_упр\)
    3. По закону Гука \(F_упр=F=k\times\Delta l\) отсюда выводим формулы для нахождения удлинения пружин: \(\Delta l_1=\frac F{k_1}\) и \(\Delta l_2=\frac F{k_2}\)
    4. Подставляем известные нам числовые значения в формулы и получаем ответ: \(6 см\) — удлинение первой пружины, \(5 см\) — удлинение второй пружины.2.\)
    5. Переводим минуты в часы: \(1\) минута=\(60\) секунд.
    6. Найти работу можно по формуле: \(A=F\times S\)
    7. В данных условиях \(S=h\), а \(F=g\times m\)
    8. В условиях задачи нет значения массы тела, но мы помним, что массу можно найти по формуле: \(m=p\times V\)
    9. Формула нахождения работы приобретает следующий вид: \(A=p\times V\times g\times h\)
    10. Подставляем известные числовые значения в формулу и получаем ответ: работа = 3 528 000 000 Дж = 3 528 МДж. 
    Закон сохранения энергии и импульса

    Задача:

    Тепловоз массой 130 тонн приближается со скоростью 2 м/с к неподвижному составу массой 1170 тонн. С какой скоростью будет двигаться состав после сцепления с тепловозом? 

    Решение:

    1. Записываем известные нам данные: \(m_1=130\) тонн, \(V_1=2\) м/с, \(m_2=1170\) тонн,  \(V_2=0\) м/с, V=?
    2. Согласно закону сохранения импульса \(m_1\times V_1+m_2\times V_2=m_3\times V_3\)
    3. Из этой формулы получаем уравнение для нахождения скорости состава после сцепления: \(V_3=\frac{m_1\times V_1}{m_1+m_2}\)
    4. Подставляем известные нам значения в формулу и получаем искомую скорость, равную \(0,2\) м/с.

    Статика



    Источник: infourok.ru

    Статика — третий раздел механики, который изучает механические системы в условиях равновесия и действие приложенных к ним сил.

    Для решения задач по статике необходимо обязательно рисовать схемы, иллюстрирующие заданные процессы, определять модули и направления сил, пользоваться законами сопротивления материалов.

    Статика включает в себя следующие разделы:

    • равновесие тел;
    • давление в жидкостях и газе;
    • закон Архимеда.
    Равновесие тел



    Источник: infourok.ru

    Давление в жидкостях и газе

    Задача:

    Водолаз в жестком скафандре может погружаться на глубину 250 метров, искусный ныряльщик — на 20 метров. Определите давление воды в море на этих глубинах.

    Решение:

    1. Записываем известные нам данные из условия задачи: \(h_1\)=250 м, \(h_2\) =20 м, \(p=1030 кг/м^3\), \(g=9,8\) Н/кг, \(p_1=?,\) \(p_2=?\)
    2. По формуле \(P_1=p\times g\times h_1\) определяем давление воды для водолаза, оно будет равно примерно 2524 кПа.3.\)

    Молекулярная физика

    Молекулярная физика — это один из разделов физики, описывающий физические свойства объектов путем изучения их молекулярного строения.

    В основе всех задач по молекулярной физике лежит уравнение молекулярно-кинетической теории: \(P=\frac13\times m_0\times n\times V_2\)



    Источник: znakka4estva.ru

    Термодинамика



    Источник: present5.com

    Термодинамика — физический раздел, который изучает общие свойства макроскопических систем, способы передачи и превращения энергии в них.

    В раздел термодинамики входят следующие темы:

    • теплота сгорания топлива; 
    • изменение внутренней энергии тела при совершении работы; 
    • внутренняя энергия идеального газа;
    • первый закон термодинамики;
    • КПД теплового двигателя.
    Теплота сгорания топлива

    При решении задач на сгорание топлива, важно помнить про удельную теплоту сгорания каждого вида топлива.7\) Дж/кг, \(Q=?\)

  • По формуле \(Q=q\times m\) определяем теплоту сгорания и получаем 95 кДж.
  • Изменение внутренней энергии тела при совершении работы

    Задача:

    Вычислите внутреннюю энергию 1 килограмма воды при ее нагревании на 2 Кельвина.

    Решение:

    1. Записываем известные и неизвестные величины из условий задачи: \(m=1\)  кг, \(T=2\)К, \(U=?\), не забываем про удельную теплоемкость воды \(c=4200\) Дж/кгхК.
    2. Количество теплоты, которое получит вода, будет затрачено на изменение ее внутренней энергии, т.е. \(U=Q\).
    3. \(Q=c\times m\times T\) следовательно, \(U=c\times m\times T\)
    4. Подставляем числовые значения в формулу и получаем ответ: 8400 Дж.
    Внутренняя энергия идеального газа, первый закон термодинамики

    При решении таких задач важно помнить про молярную массу вещества и универсальную газовую постоянную.

    Задача:

    Чему будет равна внутренняя энергия гелия массой 200 грамм при условии, что температура будет увеличена на 20 Кельвинов? 

    Решение:

    1. Фиксируем известные величины: \(m=200\) г, \(\Delta T= 20\) К.3\).
    2. Коэффициент полезного действия определяется по формуле: \(\eta=\frac{A_п}{A_з}\)
    3. \(A_п\) равна количеству теплоты (\(Q\)), которое необходимо для изменения температуры воды. \(A_п=Q=c\times m\times T.\) Массу воды найдем по формуле: \(m_1=p_1\times V_1\)
    4. \(A_з\) равна количеству теплоты, выделенному при сгорании керосина массой 80 грамм, следовательно, \(A_з=q\times m_2\)
    5. Подставив все известные величины в формулу, получаем ответ: КПД = 0,33.

    Электростатика



    Источник: infourok.ru

    Электростатика — это раздел физики об электричестве, который изучает взаимодействие электрических зарядов, находящихся в неподвижности.

    К задачам по электростатике относятся задачи на :

    • закон Кулона; 
    • напряженность и работу электростатического поля; 
    • электроемкость.
    Закон Кулона

    Задача:

    Определите силу взаимодействия двух одинаковых точечных зарядов по 1 микро кулону, которые находятся на расстоянии 30 сантиметров друг от друга.2/м\).

  • Согласно закону Ома \(I=\frac UR\) отсюда \(U=U=I\times R\)
  • Сопротивление определяем по формуле: \(R=p\times\frac lS\)
  • Подставляем числовые данные, находим сопротивление. Оно равно 0,34 Ом.
  • Находим значение напряжения: 1,7 В.
  • Задача на работу и мощность тока:

    Определите мощность и работу электродвигателя вентилятора за 10 минут, если при напряжении 220 Вольт сила тока в электродвигателе составила 1 Ампер.

    Решение:

    1. Записываем условия: \(t=10\) мин, \(U= 220\) В, \(I=1 А\), \(P=?\) \(A=?\)
    2. Переводим минуты в секунды, получаем 600 секунд.
    3. По формуле \(P=I\times U\) определяем мощность тока. Она равна 220 Вт.
    4. По формуле \(A=P\times t\) находим работу, получаем 132000 Дж или 132 кДж.
    Магнитное поле

    К задачам раздела «Магнитное поле» относятся задания на:

    • силу Ампера;
    • силу Лоренца; 
    • магнитный момент, индукцию и самоиндукцию, энергию магнитного поля.-15 Н\).
    • Задача на магнитный поток и ЭДС индукции:



      Источник: kopilkaurokov.ruИсточник: kopilkaurokov.ru

      Колебания и волны



      Источник: prezentacii.org

      В разделе физики «Колебания и волны» изучают следующие темы:

      • механические гармонические колебания математических маятников;
      • пружинный маятник; 
      • энергия механических колебаний; 
      • механические волны; 
      • колебательный контур;
      • электромагнитные волны.

      Задача на колебания математического маятника:



      Источник: videouroki.net

      Задача на пружинный маятник:



      Источник: znanio.ru

      Задача на колебательный контур:



      Источник: mypresentation.ru

      Для того, чтобы задания по физике решались совсем легко, предмет нужно полюбить. Если это не про вас, не переживайте! Посвящайте свое время любимым дисциплинам и хобби, а физику оставьте для профессионалов Феникс.Хелп.

      Онлайн физика — Теория, задачи по физике и решения

      Физика — фундаментальная наука, изучение которой сегодня стало более доступным и увлекательным, благодаря достижениям современных информационных технологий.

      Особую актуальность изучения предмета онлайн добавил эпидемиологический фактор: в условиях самоизоляции и противовирусных ограничений подготовка к ОГЭ и ЕГЭ по физике становится отличным выбором для тех, кто готовится к продолжению образования в высших учебных заведениях и испытывает необходимость в дополнительных занятиях для получения высокого результата на ЕГЭ по физике

      Если вам нужен опытный репетитор по физике онлайн (online), или если вы хотите проверить свои знания и умения решать задачи по ЕГЭ по физике в тестах, наш сайт вам поможет!

      Онлайн курс подготовки к ЕГЭ по физике

      Внимание! Вскоре на сайте будет доступен онлайн курс подготовки к ЕГЭ по физике. Курс будет включать видео-уроки, проверочные тесты, домашние задания и разбор задач. Курс будет доступен в дистанционном формате. Для его прохождения вам понадобится лишь компьютер, телефон или планшет, подключенный к интернету. Курс будет доступен 24/7, вы можете выбирать любое удобное для вас время и темп прохождения заданий. 

      Физика онлайн обеспечивает не только относительно лёгкое усвоение знаний, но и позволяет сформировать умения и навыки, столь необходимые для успешного освоения предмета. Наглядность онлайн-занятий обеспечивает использование интерактивной доски, что даёт хорошие результаты при усвоении новых знаний и приобретения навыков решения задач, что делает более доступным понимание и усвоение методов решения задач различного уровня сложности. Физика онлайн предоставляет доступ к теории, входящей в действующую «Программу по физике  для средней  (полной) общеобразовательной школы», а также позволяет смотреть все разделы физики и пройти тренировочные тесты по основным разделам курса физики в процессе подготовки, тут же получив результаты, и сверить своё решение с верным решением, начать отрабатывать имеющиеся пробелы в знаниях и совершенствовать навыки решения задач.

      Задачи по физике и решения вы найдете в наших тренировочных тестах по разделам:

      Решая физические задачи | Безграничная физика

      Анализ размеров

      Любая физическая величина может быть выражена как произведение комбинации основных физических размеров.

      Цели обучения

      Расчет преобразования одного вида измерения в другой

      Основные выводы

      Ключевые моменты
      • Размерный анализ — это практика проверки отношений количества физических величин путем определения их размерностей.
      • Часто приходится сталкиваться с проблемой, когда для выражения одной и той же базовой величины используются разные измерения. Следующее уравнение можно использовать для определения коэффициента преобразования между двумя производными размерами: [латекс] {\ text {n} _2 = \ frac {\ text {u} _2} {\ text {u} _1} * \ text { n} _1} [/ латекс].
      • Размерный анализ также можно использовать как простую проверку вычислений, теорий и гипотез.
      Ключевые термины
      • измерение : мера пространственной протяженности в определенном направлении, например высота, ширина или ширина или глубина.

      Размеры

      Размерность физической величины указывает, как она соотносится с одной из семи основных величин. Этими основными величинами являются:

      .

      • [M] Масса
      • [L] Длина
      • [T] Время
      • [A] Ток
      • [K] Температура
      • [моль] Количество вещества
      • [cd] Сила света

      Как видите, символ заключен в пару квадратных скобок. Это часто используется для представления измерения отдельной базовой величины.{-1}] [/ латекс]. Любая физическая величина может быть выражена как произведение комбинации основных физических размеров.

      Анализ размеров

      Размерный анализ — это практика проверки отношений между физическими величинами путем определения их размеров. Измерение любой физической величины — это комбинация основных физических измерений, составляющих ее. Анализ размеров основан на том факте, что физический закон не должен зависеть от единиц измерения физических переменных.Его можно использовать для проверки достоверности полученных уравнений, вычислений и гипотез.

      Расчетные размеры

      Измерения производных величин могут включать несколько или все измерения в отдельных основных величинах. Чтобы понять технику записи размеров производной величины, мы рассмотрим случай силы. Сила определяется как:

      [латекс] \ small {\ text {F} = \ text {m} \ cdot \ text {a} \\\ text {F} = [\ text {M}] [\ text {a}]} [/ латекс]

      Размер ускорения, представленный как [a], сам по себе является производной величиной, являющейся отношением скорости и времени.{-2}]} [/ латекс]

      Преобразование размеров

      На практике может потребоваться преобразование одного измерения в другое. Что касается обычных преобразований, вы, возможно, уже знаете, как преобразовать их с головы до ног. Но для менее распространенных полезно знать, как найти коэффициент преобразования:

      [латекс] \ small {\ text {Q} = \ text {n} _1 \ text {u} _1 = \ text {n} _2 \ text {u} _2} [/ latex]

      , где n представляет количество на u измерений. Затем вы можете использовать коэффициенты, чтобы вычислить преобразование:

      [латекс] \ displaystyle \ small {\ text {n} _2 = \ frac {\ text {u} _2} {\ text {u} _1} \ cdot \ text {n} _1} [/ latex]

      Тригонометрия

      Тригонометрия играет центральную роль в использовании диаграмм свободных тел, которые помогают визуально представлять сложные физические задачи.

      Цели обучения

      Объясните, почему тригонометрия полезна для определения горизонтальных и вертикальных составляющих сил

      Основные выводы

      Ключевые моменты
      • Важно определить проблему и неизвестные и нарисовать их на диаграмме свободного тела.
      • Законы косинуса и синуса можно использовать для определения вертикальных и горизонтальных составляющих различных элементов диаграммы.
      • В диаграммах свободного тела используются геометрия и векторы для визуального представления физических задач.
      Ключевые термины
      • тригонометрия : Раздел математики, который занимается отношениями между сторонами и углами треугольников и вычислениями на их основе, особенно тригонометрическими функциями.

      Тригонометрия и решение физических задач

      В физике большинство задач решаются гораздо легче, если использовать диаграмму свободного тела. Диаграммы свободного тела используют геометрию и векторы для визуального представления проблемы.Тригонометрия также используется для определения горизонтальных и вертикальных составляющих сил и объектов. Диаграммы свободного тела очень помогают визуально определить, какие компоненты неизвестны и где применяются моменты. Они могут помочь проанализировать проблему, статическую или динамическую.

      Когда люди рисуют схемы свободного тела, часто не все идеально параллельно и перпендикулярно. Иногда людям необходимо проанализировать горизонтальные и вертикальные составляющие сил и ориентацию объекта.Когда сила или объект не действует параллельно оси x или y , люди могут использовать базовую тригонометрию, чтобы использовать простейшие компоненты действия для его анализа. В принципе, все следует рассматривать с точки зрения x и y , что иногда требует некоторых манипуляций.

      Free Body Diagram : Удочка прикреплена к стене шарнирно и удерживается на веревке.

      Стержень «AB» прикреплен к стене на шарнире «A» и удерживается неподвижно с помощью веревки, как показано на.Это упражнение предполагает рисование схемы свободного тела. Чтобы упростить задачу, сила F будет выражена через ее горизонтальную и вертикальную составляющие. Удаление всех остальных элементов из изображения помогает создать законченную схему свободного тела.

      Схема свободного тела : Схема свободного тела в виде готового продукта

      Имея готовую диаграмму свободного тела, люди могут использовать свои знания тригонометрии и законов синуса и косинуса для математического и численного представления горизонтальных и вертикальных компонентов:

      Общие приемы решения проблем

      Диаграммы свободного тела используют геометрию и векторы для визуального представления проблемы.

      Цели обучения

      Построить диаграмму свободного тела для физического сценария

      Основные выводы

      Ключевые моменты
      • Свободная диаграмма тела позволяет визуально выделить проблему, которую вы пытаетесь решить, и упростить ее до простой геометрии и тригонометрии.
      • При рисовании этих диаграмм полезно рисовать только само тело и силы, действующие на него.
      • Рисование других объектов и внутренних сил может сжать диаграмму и сделать ее менее полезной.
      Ключевые термины
      • статический : фиксируется на месте; без движения.
      • динамический : Смена; активный; в движении.

      В физике большинство задач решаются гораздо легче, если использовать диаграмму свободного тела. При этом используются геометрия и векторы для визуального представления проблемы, а тригонометрия также используется для определения горизонтальных и вертикальных компонентов сил и объектов.

      Назначение: Диаграммы свободного тела очень полезны для визуального определения, какие компоненты неизвестны, где применяются моменты, и помогают анализировать проблему, статическую или динамическую.

      Как сделать диаграмму свободного тела

      Чтобы нарисовать диаграмму свободного тела, не беспокойтесь о ее масштабировании, это просто то, что вы используете, чтобы помочь себе определить проблемы. Сначала вы хотите смоделировать тело одним из трех способов:

      • Как частица. Эта модель может использоваться, когда любые эффекты поворота равны нулю или не представляют интереса, даже если само тело может быть удлинено. Тело может быть представлено небольшим символическим пятном, а диаграмма сокращается до набора параллельных стрелок.Сила, действующая на частицу, — это вектор , связанный с .
      • жесткий удлиненный . Напряжения и деформации не представляют интереса, в отличие от поворотных эффектов. Стрелка силы должна лежать вдоль силовой линии, но где она не имеет значения. Сила, действующая на удлиненное твердое тело, представляет собой вектор скольжения .
      • нежесткий удлиненный . Точка приложения силы становится решающей и должна быть указана на диаграмме. Сила на нежесткое тело — это вектор , связанный с .Некоторые инженеры используют конец стрелки для обозначения точки приложения. Остальные используют наконечник.

      Что можно и нельзя

      Что включать: Поскольку диаграмма свободного тела представляет само тело и внешние силы, действующие на него. Итак, вы захотите включить в диаграмму следующие элементы:

      • Тело: обычно это схематическое изображение в зависимости от тела — частичное / расширенное, жесткое / нежесткое — и на какие вопросы необходимо ответить. Таким образом, если рассматривается вращение корпуса и крутящий момент, необходимо указать размер и форму корпуса.
      • Внешние силы: указаны стрелками с метками. В полностью решенной задаче стрелка силы может указывать направление, величину точки приложения. Этими силами могут быть трение, сила тяжести, нормальная сила, сопротивление, растяжение и т. Д.

      Не включать:

      • Не показывать тела, кроме тела интереса.
      • Не показывать силы, прилагаемые телом.
      • Внутренние силы, действующие на различные части тела другими частями тела.
      • Любая скорость или ускорение не учитываются.

      Как решить любую физическую задачу : выучите пять простых шагов за пять минут! В этом выпуске мы рассказываем о наиболее эффективном методе решения проблем, с которым я столкнулся, и призываем некоторых нечетких друзей помочь нам запомнить шаги.

      Схема свободного тела : Используйте этот рисунок для решения примера проблемы.

      Решение задач в физике

      Решение задач в физике

      РЕШЕНИЕ ЗАДАЧ ФИЗИКИ
      СТРАТЕГИЯ

      Др.Марк Холлабо
      Нормандальский муниципальный колледж

      http://www.nr.cc.mn.us/physics/Faculty/HOLLABGH/probsolv.htm

      Два фактора могут помочь вам стать лучше
      решатель задач физики. Прежде всего, вы должны знать и
      понять принципы физики. Во-вторых, вы должны
      иметь стратегию применения этих принципов в новых ситуациях
      в чем может быть полезна физика. Мы называем эти ситуации
      проблемы. Многие студенты говорят: «Я понимаю
      материал, я просто не могу решать проблемы.«Если это
      верно для вас как изучающего физику, тогда, возможно, вам нужно
      развивать свои навыки решения проблем. Имея стратегию
      организация этих навыков может вам помочь.

      Решению задач физики можно научиться просто
      как вы научились водить машину, играть на музыкальном инструменте или
      кататься на велосипеде. Что может помочь вам больше всего, так это иметь
      общий подход к решению каждой проблемы, которую вы
      сталкиваться. Вы можете использовать разные инструменты или тактики с
      различных областях физики, но общая стратегия остается
      такой же.Скорее всего, вы уже приобрели
      навыки решения проблем и привычки из предыдущих курсов в
      физика, химия или математика. Как и другие области
      обучения и жизни, некоторые из этих привычек могут быть полезны и
      некоторые могут действительно помешать вашему прогрессу в изучении решения
      проблемы физики.

      Итак, изучая этот новый подход, будьте
      готовы пробовать новые идеи и отказаться от старых привычек, которые могут
      факт мешает вашему пониманию. Когда вы взрослеете как
      решатель проблем физики, вы обнаружите, что этот подход будет
      стать для вас второй натурой.Вы начнете автоматически
      делать то, что приведет вас к созданию эффективного
      Решение проблемы.

      Как и во многих других учебных мероприятиях,
      полезно разбить стратегию решения проблемы на основные и
      мелкие шаги. Стратегия, которую мы хотим, чтобы вы изучили,
      пять основных шагов: Сфокусируйтесь на проблеме , Физика
      Описание
      , Планирование решения , Выполнение плана ,
      и Оцените решение .Давайте возьмем
      подробно рассмотрите каждый из этих шагов, а затем выполните образец
      проблема следования стратегии. На этом этапе нашей
      обсуждение, не беспокойтесь, если есть физические термины или концепции
      что вы не понимаете. Вы изучите эти концепции
      по мере необходимости. Затем вернитесь к этому обсуждению.

      ФОКУСИРУЙТЕ ПРОБЛЕМУ
      Обычно, когда вы читаете заявление
      проблема физики, вы должны визуализировать задействованные объекты и
      их контекст.Вам нужно нарисовать картинку и указать любую
      предоставленная информация.

      (1) Во-первых, создайте мысленный образ проблемной ситуации.

      (2) Затем нарисуйте грубую, хотя и буквальную картинку, показывающую
      важные объекты, их движение и их
      взаимодействия. Например, взаимодействие может состоять
      один объект соединяется с другим веревкой.
      (3) Пометьте всю известную информацию. На этом этапе не
      беспокоиться о присвоении алгебраических символов конкретным
      количества.

      Иногда вопрос, задаваемый в
      проблема не очевидна. «Веревка безопасна?»
      не то, на что вы можете прямо ответить. Спросите себя, что
      конкретно спрашивают? Как это переводится в
      какое-то исчисляемое количество?

      Есть много способов решить физику
      проблема. Одна из частей обучения тому, как решать проблему, — это
      знать, какой подход использовать. Вам нужно будет обрисовать
      концепции и принципы, которые, по вашему мнению, будут полезны при решении
      проблема.


      Если задействованы простые движения, используйте кинематику
      определение скорости и ускорения.
      Если задействованы силы и объекты взаимодействуют из-за них
      сил используйте законы движения Ньютона.
      Силы, которые действуют в течение определенного промежутка времени и заставляют объекты
      изменить их скорости предлагает с помощью функции сохранения
      импульса.
      Часто в ситуациях, связанных с теплофизикой или
      электромагнетизм, принцип сохранения энергии
      является полезным.
      Возможно, вам потребуется указать временные интервалы, в течение которых
      применение каждого принципа будет наиболее полезным.
      Важно определить любые ограничения, присутствующие в
      такая ситуация, например, «машина не выезжает»
      дорога ».
      Укажите любые приближения или упрощения, которые, по вашему мнению,
      облегчит решение проблемы, но не
      существенно повлияют на результат. Часто мы
      игнорировать силы трения из-за сопротивления воздуха.

      Ваш подход, вероятно, будет очень
      последовательно на протяжении всего определенного раздела учебника.
      Задача для вас будет заключаться в том, чтобы применить этот подход в различных
      ситуаций.

      ОПИСАТЬ ФИЗИКУ
      «Физическое описание»
      проблема переводит данную информацию и очень дословно
      изображение в идеализированную диаграмму и определяет переменные, которые могут
      можно манипулировать, чтобы вычислить желаемое количество.В некотором смысле
      вы переводите буквальную ситуацию в идеализированную
      ситуация, когда вы можете применить законы физики.
      Самый большой недостаток начинающих решателей физических задач —
      пытаясь применить законы физики, то есть записать
      уравнения, прежде чем приступить к качественному анализу
      проблема. Если вы можете устоять перед искушением искать
      уравнения слишком рано в решении вашей проблемы, вы станете
      гораздо более эффективное решение проблем.

      Чтобы построить описание физики, вы должны выполнить
      следующий:

      • Переведите ваше изображение в диаграмму (и), которая дает только
        важная информация для математического
        решение. На идеализированной диаграмме люди, машины,
        а другие объекты могут стать квадратными блоками или точками.
      • Определите символ для каждой важной физической переменной на
        ваша диаграмма.
      • Обычно вам нужно нарисовать систему координат, показывающую
        + и — направления.
      • Если вы используете концепции кинематики, нарисуйте движение
        диаграмма с указанием скорости объектов и
        ускорение в определенных положениях и в определенное время.
      • Если взаимодействия важны, нарисуйте идеализированное свободное тело,
        и силовые диаграммы.
      • При использовании принципов сохранения нарисуйте
        «до», «передача» (т. е. во время),
        и диаграммы «после», чтобы показать, как система
        изменения.Сбоку от диаграмм укажите значение
        для каждой физической переменной, которую вы пометили на
        диаграммы или укажите, что она неизвестна.

      Тогда, используя вопрос, ваша физика
      описания и изложенного вами подхода, вам необходимо
      определить целевую переменную. То есть вы должны решить, что
      неизвестное количество — это то, что вы должны вычислить из своего списка
      переменные. Спросите себя, отвечает ли рассчитанное количество
      вопрос.В сложных задачах может быть больше, чем
      одна целевая переменная или несколько промежуточных переменных, которые вы
      вычислить.

      Теперь, зная целевые переменные,
      и ваш подход, вы можете собрать свой набор математических
      выражений с использованием принципов и ограничений вашего
      подход, чтобы связать физические переменные с вашим
      диаграммы. Это первый раз, когда ты действительно начинаешь смотреть
      для количественных соотношений между переменными.

      ПЛАНИРОВАТЬ РЕШЕНИЕ
      Прежде чем вы начнете вычислять
      ответ, найдите время, чтобы составить план.Обычно, когда по законам
      физика выражается в уравнении, уравнение является общим,
      универсальное заявление. Вы должны построить конкретные алгебраические
      уравнения, которые позволят вам вычислить целевую переменную.

      • Определите, как уравнения в вашем наборе инструментов могут быть
        вместе, чтобы найти вашу целевую переменную. Начните с
        уравнение, содержащее целевую переменную.
      • Определите все неизвестные в этом уравнении.
      • Найдите уравнения в вашем наборе инструментов, содержащие эти
        неизвестные.
      • Продолжайте этот процесс, пока ваши уравнения не будут содержать новых
        неизвестные.
      • Пронумеруйте каждое уравнение для удобства.
      • В настоящее время не решайте уравнения численно.

      Часто опытные специалисты по решению проблем будут
      начать с целевой переменной и работать в обратном направлении, чтобы определить
      путь к ответу. Иногда единицы помогут найти
      правильный путь. Например, если вы ищете
      скорость, вы знаете, что ваш окончательный ответ должен быть в м / с.

      У вас есть решение, если в вашем плане
      много независимых уравнений, так как есть неизвестные. Если не,
      определить другие уравнения или проверить план, чтобы убедиться, что он
      вероятно, что переменная будет сокращена из ваших уравнений.

      Если у вас такое же количество уравнений
      и неизвестные, указывают порядок, в котором следует решать уравнения
      алгебраически для целевой переменной. Обычно вы начинаете
      построение плана в конце и работа в обратном направлении
      первый шаг, то есть вы записываете уравнение, содержащее
      сначала целевая переменная.

      ВЫПОЛНИТЬ ПЛАН
      Теперь вы готовы выполнить план.

      • Выполните алгебру в порядке, указанном в плане.
      • Когда вы закончите, у вас должно получиться одно уравнение с
        ваша целевая переменная изолирована с одной стороны и известна только
        количества с другой стороны.
      • Подставьте значения (числа с единицами измерения) в это
        окончательное уравнение.
      • Убедитесь, что единицы согласованы, чтобы они отменили
        правильно.

      Наконец, вычислите числовой результат для
      целевая переменная (и). Убедитесь, что ваш окончательный ответ
      понятно человеку, который оценит ваше решение.

      Чрезвычайно важно решить
      задача алгебраически перед вставкой любого числового
      значения. Некоторые неизвестные количества могут быть отменены, и вы
      на самом деле не нужно знать их числовое значение. В
      некоторые сложные задачи может быть полезно вычислить промежуточные
      числовые результаты как проверка обоснованности вашего
      решение.

      ОЦЕНИТЬ РЕШЕНИЕ
      Наконец, вы готовы оценить свои
      отвечать. Здесь вы должны руководствоваться здравым смыслом в отношении того, как
      реальный мир работает так же, как и те аспекты физического мира
      вы узнали на уроке физики.

      • Имеют ли векторные величины и величину, и направление?
      • Может ли кто-нибудь воспользоваться вашим решением?
      • Является ли результат разумным и в пределах вашего
        опыт? Вспомните, например, что автомобили
        не езжайте по шоссе со скоростью 300 миль / час.Если
        вы кладете более прохладный предмет в горячую воду, вода остывает
        вниз, и объект нагревается.
      • Есть ли смысл в единицах измерения? Скорость не измеряется,
        например, в кг / с.
      • Вы ответили на вопрос?

      По возможности рекомендуется
      внимательно прочтите решение, особенно если оно
      оценивается вашим инструктором. Если ваша оценка предполагает
      вам, что ваш ответ неправильный или необоснованный, сделайте
      заявление на этот счет и объясните свои рассуждения.

      Дополнительная литература:

      Патриция Хеллер, Рональд Кейт и Скотт Андерсон (1992),
      Обучение решению проблем посредством совместной работы. Часть
      1. Групповое или индивидуальное решение проблем, американец.
      Журнал физики
      , Vol. 60, No. 7, pp. 627-636.

      Патрисия Хеллер и Марк Холлабо (1992), Задача обучения
      Решение через кооперативную группировку. Часть 2:
      Разработка проблем и структурирование групп, США
      Журнал физики
      , Vol.60, No. 7, pp. 637-644.

      7 самых больших вопросов по физике, на которые нет ответов

      На этом загадки не заканчиваются. Атомы, как известно, электрически нейтральны — положительный заряд протонов компенсируется отрицательным зарядом электронов, — но Линкольн говорит: «Никто не знает, почему это так».

      2. Почему гравитация такая странная?

      Нет силы более знакомой, чем гравитация — в конце концов, это то, что удерживает наши ноги на земле. И общая теория относительности Эйнштейна дает математическую формулировку гравитации, описывая ее как «искривление» пространства.Но гравитация в триллион триллионов триллионов раз слабее трех других известных сил (электромагнетизма и двух видов ядерных сил, действующих на крошечных расстояниях).

      Одна возможность — на данный момент спекулятивная — состоит в том, что помимо трех измерений пространства, которые мы замечаем каждый день, есть скрытые дополнительные измерения, возможно, «свернутые» таким образом, что их невозможно обнаружить. Если эти дополнительные измерения существуют — и если гравитация способна «просачиваться» в них, это могло бы объяснить, почему гравитация кажется нам такой слабой.

      «Может быть, гравитация так же сильна, как и эти другие силы, но она быстро растворяется, выплескиваясь в эти другие невидимые измерения», — говорит Уайтсон. Некоторые физики надеялись, что эксперименты на LHC дадут намек на эти дополнительные измерения, но пока безуспешно.

      3. Почему кажется, что время течет только в одном направлении?

      Со времен Эйнштейна физики думали, что пространство и время образуют четырехмерную структуру, известную как «пространство-время». Но пространство очень сильно отличается от времени.В космосе мы можем перемещаться, как захотим. Что касается времени, мы застряли. Мы становимся старше, а не моложе. И мы помним прошлое, но не будущее. Время, в отличие от пространства, кажется, имеет предпочтительное направление — физики называют его «стрелой времени».

      Некоторые физики подозревают, что второй закон термодинамики дает ключ к разгадке. Он утверждает, что энтропия физической системы (грубо говоря, количество беспорядка) со временем увеличивается, и физики думают, что это увеличение и определяет направление времени.(Например, разбитая чашка имеет больше энтропии, чем неповрежденная — и, конечно же, разбитые чашки всегда появляются после неповрежденных, а не раньше.)

      Энтропия может расти сейчас, потому что раньше она была ниже, но почему это низко для начала? Была ли энтропия вселенной необычно низкой 14 миллиардов лет назад, когда она возникла в результате Большого взрыва?

      Для некоторых физиков, включая Шона Кэрролла из Калифорнийского технологического института, это недостающий элемент головоломки. «Если вы скажете мне, почему ранняя Вселенная имела низкую энтропию, я смогу объяснить остальное», — говорит он.По мнению Уайтсона, энтропия — это еще не все. «Для меня, — говорит он, — самый глубокий вопрос заключается в том, почему время так отличается от пространства?» (Недавнее компьютерное моделирование, кажется, показывает, как асимметрия времени может возникать из фундаментальных законов физики, но работа противоречива, и окончательная природа времени продолжает вызывать страстные споры.)

      Связанные

      4. Откуда все взялось. антивещество идет?

      Антивещество может быть более известным в художественной литературе, чем в реальной жизни.В оригинальном «Звездном пути» антивещество реагирует с обычным веществом, приводя в движение варп-двигатель, который движет США. Предприятие на скоростях, превышающих скорость света. В то время как варп-драйв — это чистая выдумка, антивещество вполне реально. Мы знаем, что для каждой частицы обычного вещества может быть идентичная частица с противоположным электрическим зарядом. Антипротон, например, похож на протон, но с отрицательным зарядом. Между тем, античастица, соответствующая отрицательно заряженному электрону, является положительно заряженным позитроном.

      Физики создали антивещество в лаборатории. Но когда они это делают, они создают равное количество материи. Это говорит о том, что в результате Большого взрыва материя и антивещество были созданы в равных количествах. Однако почти все, что мы видим вокруг себя, от земли под ногами до самых далеких галактик, состоит из обычной материи.

      Что происходит? Почему материи больше, чем антивещества? Мы предполагаем, что Большой взрыв каким-то образом произвел немного больше вещества, чем антивещества.«Что должно было произойти в начале истории Вселенной — в самые моменты после Большого взрыва — так это то, что на каждые 10 миллиардов частиц антивещества приходилось 10 миллиардов и одна частица материи», — говорит Линкольн. «И вещество и антивещество уничтожили 10 миллиардов, оставив один. И этот маленький «единица» — это та масса, которая составляет нас ».

      Но почему вообще небольшой избыток вещества над антивеществом? «Мы действительно этого не понимаем, — говорит Линкольн. «Это странно.«Если бы начальные количества вещества и антивещества были равны, они бы полностью уничтожили друг друга в виде всплеска энергии. В этом случае, говорит Линкольн, «нас бы не было».

      Актуальные задачи физики.

      Собрано, отредактировано и дополнено Дональдом Симанеком.

      Учебные домашние задания слишком часто не бросают вызов изобретательности учащихся и не требуют проявлений проницательности и творческого подхода при их решении. Студенты часто могут их решить, подставляя числа в формулу (или две) из учебника и вытаскивая ответ, так называемый метод «заткни и пей».

      Хорошо время от времени решать проблемы, которые , а не можно решить так легко, проблемы, которые бросают вызов и разочаровывают. Вот несколько примеров. Большинство из них — старая классика, некоторые — новые.

      1. Просто катится.

      Три связанных загадки о катящихся катушках.

      1. На рис. 1 показано колесо (серое) с центральной ступицей (зеленое). Он может свободно катиться по ровной плоской поверхности стола.Балка (желтая) прижимается к нижней части ступицы и тянется вправо с силой F. Радиус большого колеса — 13, радиус ступицы — 3. Балка, ступица и колесо катятся без проскальзывания. .

      • В каком направлении большое колесо катится по столу?
      • Насколько быстро он катится относительно скорости желтого луча?

      2. На рис. 2 показана аналогичная ситуация, но балка опирается на верх ступицы.Те же вопросы.

      3. На рис. 3 показан другой фрикционный (или зубчатый) ролик, контактирующий со ступицей. Сверху на него опирается балка. Те же вопросы.

      Сравните свои ответы. Они вообще удивительны? Правильно ли вы их спрогнозировали, прежде чем действительно решили проблемы?

      4. Теперь, можете ли вы изобрести механическую тележку, которая будет двигаться в направлении, противоположном направлению приложенной силы, медленнее, чем движется агент, применяющий эту силу?

      5.Можете ли вы изобрести механическую тележку, которая будет двигаться в направлении, противоположном направлению приложенной силы, быстрее, чем движется агент, применяющий эту силу?

      В наши дни игрушки не поощряют такого рода пытливые вопросы и возня. Кто-то может возразить, что всякий, кто ездил на велосипеде и ремонтировал его, знаком с таким механизмом. Или играл с игрушкой йо-ю. Но большинство из них не задавало вопросов и проводило эксперименты, чтобы найти ответы.

      Вы можете найти механизмы аналогичного типа здесь: Прямо по ветру быстрее ветра.

      2. Ужасный резистор

      тетраэдр.

      Эта электрическая сеть содержит пять одинаковых резисторов и один резистор, который заметно отличается от других. Как вы можете идентифицировать разные резисторы, используя только омметр с наименьшим количеством измерений? Какое минимально необходимое количество измерений в худшем случае? В лучшем случае (если повезет)?
      Вы не можете отсоединять или распаивать резисторы .Вы не можете подключать ничего, кроме стандартного омметра. Опишите свой метод и рассуждения. Четыре соединения помечены для удобства.

      3. Классический резисторный куб.

      Это старенький, но все же вкусный.

      Двенадцать одинаковых резисторов сопротивления r спаяны в кубическую сеть.

      (а) Какое сопротивление измеряется омметром между А и С?

      б) Какое сопротивление между А и В?

      (c) Какое сопротивление между A и D?

      Дайте все ответы по р.

      Подсказка: эта сеть очень симметрична. Это позволяет находить проницательные решения. Есть и непростые решения. См. Трудный вопрос в «Палеотехнологии», который является хорошим упражнением в использовании преобразований Уай-Дельта. Этот веб-сайт является источником красивой графики. Но вам все это не нужно. Это как расколоть грецкий орех кувалдой. Ищите более простое решение, используя законы Кирхгофа.

      4. Классическая пирамида резисторов.

      Припаяйте шесть одинаковых резисторов размера r в форме равносторонней пирамиды.Результатом будет сеть, подобная той, которая показана в задаче 2 выше.
      Каково сопротивление пирамиды между любыми двумя переходами в единицах r?

      5. Где на Земле?

      Где на поверхности земли вы могли бы пройти 1 милю на юг, 1 милю на восток и одну милю на север и в конечном итоге оказаться в исходной точке?

      Один ответ найти легко. Но есть и другие ответы. Многие из них. Где они и сколько их?

      6.Прогулки с собакой.

      Джейн выгуливает собаку, пятно. Она видит своего друга Дика, идущего к ней по той же длинной прямой дороге. И Дик, и Джейн идут со скоростью 3 мили в час. Когда Дик и Джейн находятся на расстоянии 600 футов друг от друга, Спот бежит от Дика к Джейн, разворачивается и бежит обратно к Дику, а затем туда и обратно между ними с постоянной скоростью 8 миль в час. Дик и Джейн продолжают идти навстречу друг другу со скоростью 3 мили в час. Пренебрегая временем, потерянным каждый раз, когда Спот меняет направление, как далеко забежал Спот за то время, которое требуется Дику и Джейн, чтобы встретиться?


      Ответы и обсуждение.


      Вернитесь на страницу Дональда Симанека.
      Вернуться к началу этого документа.
      Вернитесь в главную галерею Музея неработающих устройств.

      Формулирование двух частей физических проблем | Информация для инструкторов | Вибрации и волны Решение проблем

      В этом разделе Вит Буза проводит различие между физикой и математикой, связанной с физическими проблемами. Он обсуждает, как это различие помогает ему определить, поняли ли студенты суть проблемы.

      Физические проблемы состоят из двух принципиально разных частей, и эти две части отражают суть научного метода. Первая часть относится к тому, что я называю «физическими законами в действии» (или пониманием физики). Для этого требуется, чтобы учащиеся обладали способностью переводить физические ситуации в математические выражения. Вторая часть включает в себя собственно математические вычисления. Когда у учащихся возникают проблемы с решением проблемы, очень важно, чтобы учитель определил, какая из двух частей вызывает затруднения у ученика.По моему опыту, учителя редко делают это.

      Часть I. Преобразование физических ситуаций в математические уравнения

      Физические задачи состоят из двух принципиально разных частей, и эти две части отражают суть научного метода.

      —Wit Busza

      Я могу вспомнить бесконечное количество примеров, когда студенты приходили ко мне и говорили: «Я не понимаю. Я действительно разбираюсь в физике, но мне кажется, что я никогда не могу решить проблемы. Что не так?» Ответ на это почти всегда таков: «Вы не понимаете физику.«Хотя большинство студентов считают постановку задачи тривиальной, на самом деле это самая сложная часть решения проблемы. Я часто прошу студентов сначала визуализировать механические части, увидеть, как они взаимосвязаны, и нарисовать схему того, что происходит. Вы будете удивлены, как часто они делают это неправильно! Так много учеников испытывают трудности с визуализацией в двух и трех измерениях.

      Когда я вспоминаю свой собственный опыт в начальной школе, у нас часто возникали проблемы со словами. Вы знаете , кто-то пришел с двумя яблоками и проделал с ними то-то и то-то и т. д.Иногда мне было трудно уследить за происходящим. Видите, это не математика. Это способность превращать язык в картину происходящего. И это большая проблема для студентов. Но это еще не конец.

      Теперь вы должны взять это описание ситуации на обычном языке — скажем, например, я держу мяч в руке на высоте трех метров над полом, а затем отпускаю — и перевести его в математику.

      Итак, этот мяч начал падать.Как оно падает? И на этом этапе вы, возможно, можете визуализировать это, но вам нужны законы природы, чтобы перевести это в математическую форму. Этот шар движется все быстрее и быстрее, что математически означает, что вторая производная его положения не равна 0, а является некоторой константой — в данном случае g. Итак, как только вы визуализируете картину, даже если вы можете представить, что происходит, вам все равно придется перевести это в математику. И это физика — понимание природы, того, как ведет себя система. Все это первая часть решения задачи, и, повторяю, самая сложная часть.Но большинство студентов думают, что это легкая часть, а все остальное — сложная!

      Блок-схема решения физических задач (из справочного видео по решению проблем, «Простое гармоническое движение и введение в решение проблем»).

      Помочь себе можно по-разному. Например, если это проблема механики, вы рисуете диаграмму сил. Поразительно, сколько знаний и опыта нужно иметь, чтобы нарисовать так называемую силовую диаграмму. Вы должны понимать, что такие величины, как скорость, ускорение, сила и т. Д., можно математически представить в виде векторов. Вы должны определить все относящиеся к делу части проблемы, а затем представить их в виде математических величин.

      Итак, в типичной задаче физическая ситуация сводится к уравнению плюс вещи, которые мы называем «граничными условиями» или «начальными условиями». У меня есть мяч. Я держу его в руке. И что с ним произойдет, зависит от того, когда я его отпущу. Выпущу ли я его сейчас или через час? Он будет делать то же движение, но математическое описание этого будет несколько отличаться в двух случаях.Итак, первый этап проблемы состоит из перевода физической ситуации, а также всех начальных и граничных условий в математические выражения.

      Часть II: Решение математических уравнений

      Математическое описание задачи знаменует собой конец первого этапа. Затем мы совершаем чудо: забываем, что математические уравнения и граничные условия имеют какое-то отношение к физике. Мы помещаем эти уравнения в компьютер или просим нашего дружелюбного профессора математики решить их.Нам не нужно сообщать компьютеру, что это имеет какое-то отношение к физической проблеме. Профессору нам нужно только сказать: «Послушайте, у меня есть эти уравнения с этими граничными условиями. Что произойдет со значением этого количества в зависимости от этого количества со временем? » Второй этап не имеет ничего общего с физикой! Мы вошли в мир математики. Только после того, как вы решите уравнения — и, конечно же, вы должны сделать это правильно, — вы сможете вернуться к физике. Это включает в себя знание правил определения того, что на самом деле происходит при решении ваших уравнений.

      Оценка физики больше, чем математики

      Я стараюсь ставить более 50% оценки учащегося за физику, вовлеченную в задачу.

      Студенты ненавидят это, потому что они приходят и говорят: «Послушайте, в первых двух строках я сделал несколько глупых ошибок. А потом я сделал три страницы математики, и у меня получилось. И вы дали мне только 40% оценки. Это не справедливо.» Конечно, ученик мог правильно понять всю математику, но он упустил суть проблемы.Я думаю, что очень важно, чтобы выставление оценок четко отражало то, что важно.

      Это сопротивление тому, что физика больше, чем математика, при оценивании заданий, является причиной того, что ученики ненавидят задачи с несколькими вариантами ответов, и почему в тот момент, когда вы задаете задачи с несколькими вариантами ответов, оценки резко падают. В задаче с множественным выбором вы должны понимать суть проблемы. Вы не можете просто угадать. И поэтому вы либо понимаете, какой из пяти законов природы применим, либо нет.Частичного кредита нет. Студенты, не особенно сильные в предмете, известны тем, что выживают за счет частичного зачета. Если у них хорошая память, они могут пройти курс физики, вообще не понимая физики, потому что они могут заработать частичный балл по математике и своей памяти. В вопросах с несколькими вариантами ответов память не так сильно помогает. Конечно, я предполагаю, что выбор хорошо продуман.

      Kinematics and Calculus — Problems — The Physics Hypertextbook

      Kinematics and Calculus — Problems — The Physics Hypertextbook

      Проблемы

      практика

      1. Выведите уравнения движения для постоянного рывка.
      2. Положение объекта описывается следующим полиномом от 0 до 10 с…

        с = т 3 -15 т 2 + 54 т

        , где s, — в метрах, t, — в секундах, а положительное — вперед. Определить…

        1. зависимость скорости объекта от времени
        2. Ускорение объекта как функция времени
        3. максимальная скорость объекта
        4. минимальная скорость объекта
        5. время, когда объект двигался назад
        6. раз, когда объект возвращался в исходное положение
        7. средняя скорость объекта
        8. средняя скорость объекта
      3. На приведенном ниже графике показано ускорение гидравлического лифта в четырехэтажном школьном здании как функция времени.

        График начинается с t = 0 с, когда дверь лифта закрылась на втором этаже, и заканчивается на t = 20 с, когда дверь открылась на другом этаже. Предположим, что положительные направления смещения, скорости и ускорения направлены вверх. Определять…

        1. максимальная скорость лифта
        2. Продолжительность короткого рывка лифта с центром 17,5 с.

        Нарисуйте соответствующие графики…

        1. скорость-время
        2. позиция-время

        Определять…

        1. наиболее вероятный этаж, на котором остановился лифт
      4. youtu.be / R1g07RpTPFE В один прекрасный день на неиспользуемой взлетно-посадочной полосе в аэропорту спортивный автомобиль высокого класса провел тест производительности от 0 до 400 км / ч. Его скорость изменялась согласно следующей функции …

        v = a (1 — e t / b )

        , где…

        = 128,1 м / с
        б = 13,31 с

        Ответьте на эти три связанных вопроса.

        1. Сколько времени потребовалось автомобилю, чтобы разогнаться до 400 км / ч (111,111 м / с)?
        2. Какое было его среднее ускорение во время теста?
        3. Какова теоретическая максимальная скорость автомобиля?

        Ответьте на эти три связанных вопроса.

        1. Выведите выражение для ускорения как функции времени.
        2. С каким ускорением машина начинала тест?
        3. С каким ускорением автомобиль разгонялся до 400 км / ч?

        Ответьте на эти два связанных вопроса.

        1. Выведите выражение для смещения как функции времени.
        2. Какое расстояние проехала машина при разгоне?

        После достижения целевой скорости 400 км / ч (111,111 м / с) водитель немедленно выключил двигатель и нажал на тормоза. Автомобиль полностью остановился через 9.451 с. Ответьте на эти три связанных вопроса.

        1. Какое было среднее ускорение автомобиля при остановке?
        2. Какое расстояние проехала машина при остановке?
        3. Какое общее расстояние проехала машина от старта до финиша.

      алгебраический

      1. Определите соотношение ускорения и скорости для постоянного рывка. (Ради аргументации назовем это пятым уравнением движения.)

      расчет

      1. Скорость объекта v в метрах в секунду описывается следующей функцией времени, t , в секундах в течение значительного промежутка времени …

        v = 4 т (4- т ) + 8

        Предполагая, что объект расположен в начале координат ( с, = 0 м), когда t = 0 с, определяют…

        1. положение объекта, с , как функция времени
        2. ускорение объекта, a , как функция времени
        3. максимальная скорость объекта
        4. если и когда, когда объект останавливается
        5. , если и когда объект возвращается в исходную точку ( с, = 0 м)
      2. Следующие уравнения определяют смещение как функцию времени.Выведите последующие уравнения для скорости и ускорения как функции времени. (Символы A , f , j , k , s 0 , π и τ — все константы.)
        1. с = ⅙ jt 3
        2. с = A sin (2π футов )
        3. с = с 0 e т / τ
      3. Грубая математическая модель проходки туннелей представлена ​​уравнением…

        где v — скорость туннелирования; s — длина тоннеля; а k — постоянная.

        1. Каким образом (увеличивается или уменьшается) изменяется скорость туннелирования при удлинении туннеля? Какой инженерный аспект туннелирования вызывает это изменение?
        2. Определите следующие величины как функцию времени…
          1. длина тоннеля
          2. скорость проходки
          3. туннельное ускорение

        (Это довольно сложная задача для студентов, которые только начали изучать математику.)

      4. Упрощенная модель автомобиля, ускоряющегося из состояния покоя по прямой дороге, задается следующим уравнением…

        v ( t ) = a (1- e bt )

        Где v ( t ) — мгновенная скорость автомобиля в футах в секунду, t — время в секундах, а A и b — постоянные.

        1. скорость
          1. Каковы единицы в коэффициентах a и b ?
          2. В чем физический смысл коэффициента а ?
          3. Какая скорость автомобиля при т = 0 с?
          4. Какова асимптота этой функции при t → ∞?
          5. Нарисуйте график зависимости скорости от времени. Включите значение v (0 с) и асимптоту v как t → ∞.
        2. позиция
          1. Выведите уравнение s ( t ) для мгновенного положения автомобиля как функции времени.(Убедитесь, что ваша функция имеет значение с = 0 м, когда t = 0 с.)
          2. Какова асимптота этой функции при t → ∞?
          3. Каков физический смысл наклона этой асимптоты?
          4. Нарисуйте график зависимости положения от времени. Включите значение с (0 с) и наклон асимптоты с как t → ∞.
        3. разгон
          1. Выведите уравнение a ( t ) для мгновенного ускорения автомобиля как функции времени.
          2. Какое ускорение автомобиля при т = 0 с?
          3. Какова асимптота этой функции при t → ∞?
          4. Нарисуйте график зависимости ускорения от времени. Включите значение a (0 с) и асимптоту a как t → ∞.
        4. youtu.be/9eH837Fh2Oo
          Примените эту модель к настоящему, но исключительному автомобилю — Red Victor 1. У этого автомобиля время разгона от нуля до шестидесяти составляет около одной секунды, а время четверти мили — около восьми секунд.Другими словами, пусть…

          v (1 сек) = 88 фут / сек
          с (8 секунд) = 1320 футов

          затем определите…

          1. значения коэффициентов a и b [я думаю, что это можно сделать только с помощью навороченного калькулятора.]
          2. максимальная скорость, а
          3. максимальное ускорение.
      5. Положение объекта описывается следующим полиномом за 10 с…

        с = т 3 -12 т 2 + 24 т

        , где s в метрах и t в секундах.

        Определить…

        1. зависимость скорости объекта от времени
        2. Ускорение объекта как функция времени
        3. максимальная скорость объекта
        4. минимальная скорость объекта
        5. время, когда объект двигался назад
        6. время (с), когда объект находился в исходной точке ( с = 0 м)
        7. время (с), когда объект вернулся в исходную позицию
        8. средняя скорость объекта
        9. средняя скорость объекта

      статистический

      1. лифт.txt
        Данные о времени ускорения в сопроводительном текстовом файле были записаны студентом, когда он ехал на лифте в офисном здании. Студент прошел из вестибюля на самый верхний этаж. Используйте эти данные и ваше любимое графическое приложение для решения следующих задач.
        1. Скорость
          1. Постройте график скорость-время.
          2. Определите крейсерскую скорость лифта.
        2. Рабочий объем
          1. Постройте график смещения-времени.
          2. Определите высоту здания.
          3. Оценить этажность дома.
      2. table-splits.shtml
        Сплит — это время, когда бегун достигает веховой дистанции в гонке. Например, в беге на 100 метров промежуточное время берется каждые 10 метров. Спуски для некоторых из самых быстрых спринтеров в мире указаны на прилагаемой веб-странице. Подгоните полином высокого порядка (четвертый, пятый, шестой или выше) к данным одного из этих спортсменов с помощью приложения для анализа данных.Определите скорость вашего спринтера как функцию времени, взяв производную от этого многочлена. Постройте график этой новой функции, а затем проанализируйте ее.
        1. Какова была начальная и конечная скорости бегуна?
        2. Какая была максимальная скорость бегуна и когда это произошло?
        3. Какая была средняя скорость бегуна?
        4. Скорость бегуна увеличилась, уменьшилась или осталась примерно такой же ближе к концу забега?
        5. Как вы думаете, насколько хорошо этот график описывает реальную производительность бегуна? Есть ли на графике проблемные участки? Как можно изменить функцию, чтобы улучшить посадку?
      3. таблица-табло.shtml
        Любительский дрэг-рейсинг открыт для всех, у кого есть уличное транспортное средство (автомобиль, легкий грузовик или мотоцикл), действующие водительские права, страховка, топливо и достаточно денег, чтобы покрыть регистрационный сбор. Он популярен в США, Великобритании и Австралии. Гонки проходят на прямой ровной трассе длиной четверть мили. В конце забега каждому участнику выдается небольшой бумажный «листок времени» с данными, собранными во время забега. Данные варьируются от места к месту, но почти всегда присутствуют следующие элементы.
        • Время реакции (R / T) — это время между сигналом о запуске и моментом, когда водитель фактически заставляет автомобиль двигаться вперед.
        • Истекшее время (ET) — это разбиения, записанные в нескольких позициях. Истекшее время начинается с момента пересечения автомобилем линии старта, а не с момента подачи сигнала о старте (как это делается в легкой атлетике).
        • Мгновенная скорость измеряется на мили (на полпути) и ¼ мили (на финише). Мы не будем использовать этот номер для этой деятельности.

        На веб-странице, которая сопровождает эту проблему, есть ссылки на изображения 50 различных временных бюллетеней — выборка из тысяч, собранных энтузиастами во время перетаскивания.com.

        1. Выберите однократный промах и перенесите информацию в таблицу, как показано ниже.
        2. Добавьте время реакции к истекшему времени, чтобы получить время гонки. (Я придумал этот термин. Я не знаю, как он на самом деле называется.)
          Необработанные данные об увольнении во времени для дрэг-рейсинга
          расстояние осталось в машине время (с) правый автомобиль время (с)
          (фут) (миль) (м) Прошло гонка Прошло гонка
          0000 0 000 0 0
          0060 018
          0330 101
          0660 201
          1000 305
          1320 ¼ 402
        3. Постройте данные о расстоянии-времени для каждой машины.(Сделайте два графика.)
        4. Выполните следующую аппроксимацию кривой на каждом графике…
          1. линейный
          2. квадратичный
          3. куб.
        5. Заполните следующую таблицу. Обязательно укажите в ответах правильные единицы. Поскольку вы сделали три разных подбора кривой, некоторые величины можно найти более чем одним методом. В разделе «Методология» укажите, какая функция (линейная, квадратичная, кубическая), какой коэффициент ( t 0 , t 1 , t 2 , t 3 ), сколько масштабирования (× 2, × 3, × 4,…, ÷ 2, ÷ 3, ÷ 4,…) потребовалось, чтобы получить ответ.
          количество методология левая машина правый вагон
          средняя скорость
          начальная скорость
          среднее ускорение
          начальное ускорение
          средний рывок
          Дрэг-рейсинг Time Slip — Анализ
      4. мустанг.txt
        В 2016 году журнал Road & Track (платная ссылка) протестировал восемь очень дорогих и очень быстрых автомобилей, чтобы определить автомобиль года с высокими характеристиками. Данные испытания на ускорение для Ford Mustang Shelby GT350R 2016 года приведены в прилагаемом текстовом файле с разделителями-табуляторами. (Мустанг не получил награду в том году.) Поскольку данные были собраны в Соединенных Штатах, основные скорости были выбраны кратными 10 милям в час. Для вашего удобства эти скорости были преобразованы в единицы СИ.
        1. Используя ваше любимое приложение для анализа данных, создайте диаграмму разброса скорости (в метрах в секунду) в зависимости отвремени (в секундах) и добавьте соответствующую кривую наилучшего соответствия.
        2. Используйте результаты аппроксимации кривой и свои знания в области вычислений, чтобы создать уравнения для…
          1. ускорение как функция времени
          2. Смещение как функция времени
        3. Во время этого теста…
          1. Расстояние, пройденное автомобилем, увеличилось, уменьшилось или осталось прежним?
          2. Скорость автомобиля увеличилась, уменьшилась или осталась прежней?
          3. Ускорение автомобиля увеличилось, уменьшилось или осталось прежним?
      5. зарм-кинематика.txt
        Данные в сопровождающем текстовом файле с разделителями табуляцией дают мгновенную скорость вертикально установленного поршня, используемого для запуска снарядов, как функцию времени. Используйте этот набор данных и ваше любимое приложение для анализа данных для решения следующих задач.
        1. Используя предоставленные данные, создайте график скорость-время и определите…
          1. максимальная скорость и
          2. средняя скорость поршня при его ускорении вверх
        2. Определите ускорение поршня как функцию времени, создайте график ускорения-времени и определите…
          1. максимальное ускорение и
          2. среднее ускорение поршня при его ускорении вверх
        3. Определите смещение поршня как функцию времени, создайте график смещения-времени и определите…
          1. окончательное смещение поршня после его остановки

        Данные адаптированы из Кампена, Качмарчика и Рата; 2006 г.

      Нет постоянных условий.

      1. Механика
        1. Кинематика
          1. Движение
          2. Расстояние и перемещение
          3. Скорость и скорость
          4. Разгон
          5. Уравнения движения
          6. Свободное падение
          7. Графики движения
          8. Кинематика и вычисления
          9. Кинематика в двух измерениях
          10. Снаряды
          11. Параметрические уравнения
        2. Dynamics I: Force
          1. Силы
          2. Сила и масса
          3. Действие-реакция
          4. Масса
          5. Динамика
          6. Статика
          7. Трение
          8. Силы в двух измерениях
          9. Центростремительная сила
          10. Кодовые рамки
        3. Энергия
          1. Работа
          2. Энергия
          3. Кинетическая энергия
          4. Потенциальная энергия
          5. Сохранение энергии
          6. Мощность
          7. Простые станки
        4. Dynamics II: Импульс
          1. Импульс и импульс
          2. Сохранение импульса
          3. Импульс и энергия
          4. Импульс в двух измерениях
        5. Вращательное движение
          1. Кинематика вращения
          2. Инерция вращения
          3. Вращательная динамика
          4. Вращательная статика
          5. Угловой момент
          6. Энергия вращения
          7. Прокат
          8. Вращение в двух измерениях
          9. Сила Кориолиса
        6. Движение планет
          1. Геоцентризм
          2. Гелиоцентризм
          3. Вселенская гравитация
          4. Орбитальная механика I
          5. Гравитационная потенциальная энергия
          6. Орбитальная механика II
          7. Плотность вытянутых тел
        7. Периодическое движение
          1. Пружины
          2. Простой генератор гармоник
          3. Маятники
          4. Резонанс
          5. Эластичность
        8. Жидкости
          1. Плотность
          2. Давление
          3. Плавучесть
          4. Расход жидкости
          5. Вязкость
          6. Аэродинамическое сопротивление
          7. Режимы потока
      2. Теплофизика
        1. Тепло и температура
          1. Температура
          2. Тепловое расширение
          3. Атомная природа материи
          4. Закон о газе
          5. Кинетико-молекулярная теория
          6. Фазы
        2. Калориметрия
          1. Явное тепло
          2. Скрытое тепло
          3. Химическая потенциальная энергия
        3. Теплопередача
          1. Проводимость
          2. Конвекция
          3. Радиация
        4. Термодинамика
          1. Тепло и работа
          2. Диаграммы давление-объем
          3. Двигатели
          4. Холодильники
          5. Энергия и энтропия
          6. Абсолютный ноль
      3. Волны и оптика
        1. Волновые явления
          1. Природа волн
          2. Периодические волны
          3. Интерференция и суперпозиция
          4. Интерфейсы и барьеры
        2. Звук
          1. Природа звука
          2. Интенсивность
          3. Эффект Доплера (звук)
          4. Ударные волны
          5. Дифракция и интерференция (звук)
          6. Стоячие волны
          7. ударов
          8. Музыка и шум
        3. Физическая оптика
          1. Природа света
          2. Поляризация
          3. Эффект Доплера (светлый)
          4. Черенковское излучение
          5. Дифракция и интерференция (свет)
          6. Тонкопленочная интерференция
          7. Цвет
        4. Геометрическая оптика
          1. Отражение
          2. Преломление
          3. Зеркала сферические
          4. Сферические линзы
          5. Аберрация
      4. Электричество и магнетизм
        1. Электростатика
          1. Электрический заряд
          2. Закон Кулона
          3. Электрическое поле
          4. Электрический потенциал
          5. Закон Гаусса
          6. Проводников
        2. Электростатические приложения
          1. Конденсаторы
          2. Диэлектрики
          3. Батареи
        3. Электрический ток
          1. Электрический ток
          2. Электрическое сопротивление
          3. Электроэнергия
        4. Цепи постоянного тока
          1. Резисторы в цепях
          2. Батареи в цепях
          3. Конденсаторы в цепях
          4. Правила Кирхгофа
        5. Магнитостатика
          1. Магнетизм
          2. Электромагнетизм
          3. Закон Ампера
          4. Электромагнитная сила
        6. Магнитодинамика
          1. Электромагнитная индукция
          2. Закон Фарадея
          3. Закон Ленца
          4. Индуктивность
        7. Цепи переменного тока
          1. Переменный ток
          2. RC-цепи
          3. Цепи RL
          4. Цепи LC
        8. Электромагнитные волны
          1. Уравнения Максвелла
          2. Электромагнитные волны
          3. Электромагнитный спектр
      5. Современная физика
        1. Относительность
          1. Пространство-время
          2. Масса-энергия
          3. Общая теория относительности
        2. Quanta
          1. Излучение черного тела
          2. Фотоэффект
          3. Рентгеновские снимки
          4. Антиматерия
        3. Волновая механика
          1. Волны материи
          2. Атомарные модели
          3. Полупроводники
          4. Конденсированное вещество
        4. Ядерная физика
          1. Изотопы
          2. Радиоактивный распад
          3. Период полураспада
          4. Энергия связи
          5. Деление
          6. Fusion
          7. Нуклеосинтез
          8. Ядерное оружие
          9. Радиобиология
        5. Физика элементарных частиц
          1. Квантовая электродинамика
          2. Квантовая хромодинамика
          3. Квантовая динамика вкуса
          4. Стандартная модель
          5. За пределами стандартной модели
      6. Фундаменты
        1. шт.
          1. Международная система единиц
          2. Гауссова система единиц
          3. Англо-американская система единиц
          4. Разные единицы
          5. Время
          6. Преобразование единиц
        2. Измерение
          1. Значащие цифры
          2. По порядку величины
        3. Графики
          1. Графическое представление данных
          2. Линейная регрессия
          3. Подгонка по кривой
          4. Исчисление
        4. Векторы
          1. Тригонометрия
          2. Сложение и вычитание векторов
          3. Векторное разрешение и компоненты
          4. Умножение вектора
        5. ссылку
          1. Специальные символы
          2. Часто используемые уравнения
          3. Физические константы
          4. Астрономические данные
          5. Периодическая таблица элементов
          6. Люди в физике
      7. Назад дело
        1. Предисловие
          1. Об этой книге
        2. Связаться с автором
          1. гленнелерт.нас
          2. Behance
          3. Instagram
          4. Твиттер
          5. YouTube
        3. Аффилированные сайты
          1. hypertextbook.com
          2. midwoodscience.org

      Практические задачи: кинематические решения — Physics-prep.com

      Практические задачи: кинематические решения

      1. (легко) Насколько быстро будет двигаться объект (движущийся по оси x) в момент времени t = 10 с, если он имел скорость 2 м / с в момент t = 0 и постоянное ускорение 2 м / с. 2 ?
      v = v o + при
      v = 2 + 2 (10)
      v = 22 м / с

      2.(легко) Автомобиль катится к обрыву с начальной скоростью 15 м / с. Максимальное отрицательное ускорение, которое могут обеспечить тормоза, составляет -0,3 м / с 2 . Если обрыв находится на расстоянии 350 м от исходного положения машины, будет ли машина преодолевать обрыв?
      Для остановки конечная скорость должна быть равна нулю до того, как автомобиль пройдет 350 м.
      v 2 = v o 2 + 2a (x — x o )
      0 = 15 2 + 2 (-0,3) (x — 0)
      x = 375 м
      Таким образом, машина не может вовремя остановиться и действительно съезжает с обрыва.

      3. (умеренный) Что больше всего повлияет на смещение объекта, который равномерно ускоряется при одномерном движении: удвоение начальной скорости или удвоение времени ускорения? Кроме того, влияет ли величина ускорения на разницу между этими двумя параметрами?
      Анализ второго кинематического уравнения, x — x o = v o t + ½at 2 , показывает, что удвоение либо времени, либо начальной скорости увеличит смещение, но поскольку время очевидно в обеих частях раствора, это будет иметь наибольший эффект.Величина ускорения смягчает влияние времени. Большее ускорение указывает на то, что временной эффект будет больше.

      4. (средняя) Тележка A движется с постоянной скоростью мимо точки 1 по прямой дороге со скоростью 0,3 м / с. Тележка B движется мимо точки 1 со скоростью 0,1 м / с, но равномерно ускоряется со скоростью 0,1 м / с 2 . Точка 2 находится на расстоянии 1,0 м от точки 1. Какая тележка первой попадает в точку 2?
      Найдите время для каждой тележки, чтобы добраться до точки 2:
      Тележка A:
      x — x o = v o t + ½at 2
      1.0 — 0 = 0,3t + 0
      t = 3,3 с
      Тележка B:
      x — x o = v o t + ½at 2
      1,0 — 0 = 0,1t + ½ (0,1) t 2
      Используйте квадратное уравнение для определения t
      t = 3,6 с
      Тележка A первой прибывает в точку 2.

      5. (легко) Маленький мяч вылетает из окна в момент t = 0. В предположении условий свободного падения, как далеко он пролетит за 2,8 секунды? Если бы мяч имел большую массу, упал бы он на большее расстояние?
      Предположим, y o = 0
      y — y o = v o t + ½gt 2
      y — 0 = 0 + -4.9 (2,8) 2
      y = -38,4 м

      Это означает, что мяч упал на 38,4 м ниже своего исходного положения.
      Ускорение свободного падения не зависит от массы, поэтому расстояние, на которое упадет более массивный объект, будет таким же.

      6. (средний) Три сферы удерживаются в разных местах над столом. Сфера 1 находится ближе всего к столу, сфера 3 — дальше всего от стола. Предположим, что все столкновения со столом абсолютно эластичны. То есть при ударе энергия не теряется, и сфера возвращается к своей исходной высоте перед тем, как снова упасть, как если бы ее уронили из состояния покоя.Считайте движение вверх от стола положительным.

      Для вопросов A и B предположим следующее:

      Начальная высота сферы 1 = 2,0 м

      Начальная высота сферы 2 = 3,0 м

      Начальная высота сферы 3 = 4,0 м

      Сфера 1 выходит из состояния покоя.

      A. С какой скоростью сфера 2 должна первоначально двигаться вверх, если она сначала ударяется о стол в то же время, когда сфера 1 ударяется о стол во второй раз?
      Время первого удара сферы 1:
      x — x o = v o t + ½at 2
      -2 = 0 — 4.9t 2
      t = 0,64 с
      Для второго удара (вниз, вверх, вниз) требуется 3 (0,64 с) = 1,92 с. — x o = -3 = v o t + ½gt 2
      -3 = v o (1,92) — 4,9 (1,92) 2
      v o = 7,8 м / с

      B. С какой скоростью сфера 3 должна первоначально двигаться вниз, если она впервые ударяется о стол в то же самое время, когда сфера 1 впервые ударяется о стол.
      x — x o = -4 = v o t + ½gt 2 = v o (0,64) — 4,9 (0,64) 2
      v o = -3,1 м / с
      Это означает, что сфера имеет скорость 3,1 м / с.

      C. Теперь предположим, что начальная скорость вверх сферы 2 составляет 1 м / с, а начальная скорость вверх сферы 3 равна 2 м / с, определите разницу в начальных высотах сфер 2 и 3, если они попадут в стол для первый раз, когда сфера 1 в шестой раз попадает в стол. Сфера 1 выходит из состояния покоя.
      Время, когда сфера 1 коснется шестого раза = 11 (0,64) = 7,0 с
      Для сферы 2:
      x — x o = v o t + ½gt 2 = 1 (7) — 4,9 (7 ) 2
      x — x o = -233 м (это означает, что он должен упасть так далеко, чтобы удариться о стол)
      Для сферы 3:
      x — x o = v o t + ½gt 2 = 2 (7) -4,9 (7) 2
      x — x o = -226 м (это означает, что он должен упасть так далеко, чтобы удариться о стол)
      Разница в двух начальных высотах равна 7 м.

      7. (легко) Тележка находится в точке x = 5 м в момент времени t = 0. Тележка ускоряется со скоростью 4 м / с 2 . Если скорость тележки при t = 0 составляет 3 м / с, найдите положение тележки при t = 2 с, а также определите, где находится тележка, когда она достигает скорости 5 м / с.
      x — x o = v o t + ½at 2
      x — 5 = 3 (2) + ½ (4) (2) 2
      x = 19 м
      v 2 = v o 2 + 2a (x — x o )
      5 2 = 3 2 + (2) (4) (x — 5)
      x = 7 м

      8.(умеренно) Автомобиль, движущийся со скоростью 20 м / с, проезжает угол улицы. Автомобиль поддерживает эту скорость даже при ограничении скорости 10 м / с. Сидевшая на углу полицейская машина начинает преследовать машину, набирая скорость 2 м / с 2 . Сколько времени понадобится полицейской машине, чтобы догнать спидер? Как далеко от угла находится точка наверстывания упущенного? Как быстро будет двигаться полицейская машина в это время?
      Для автомобиля: x = 20t
      Для полицейской машины: x = ½ (2) t 2
      В точке догонения: 20t = ½ (2) t 2
      Следовательно, t = 20 сек.
      позиция в захвате вверх: x , догоняющий = 20t = 20 (20) = 400 м
      Скорость в догоняющем режиме: v = v o + at = 0 + (2) (20) = 40 м / с

      9.(жестко) Две сферы катятся навстречу друг другу. При t = 0 сфера 1 находится в точке x = 0 и имеет скорость 10 м / с вправо, в то время как сфера 2 движется на 2 м / с влево и имеет начальное положение x = 1000 м. Наблюдения за сферами показывают следующие данные: — Через 2 секунды сфера 2 набрала скорость и движется со скоростью 10 м / с влево. Это ускорение сохраняется до столкновения сфер.
      — Видно, что сфера 1 имеет ускорение 2 м / с 2 вправо. Как быстро каждая сфера будет двигаться при столкновении?

      Найдите ускорение сферы 2:
      v = v o + at = -10 = -2 + a (2)
      a = -4 м / с 2
      Найдите положение во время удара (t) :
      Для сферы 1: x 1 = v o t + ½at 2
      x 1 = 10t + ½ (2) t 2 = 10t + t 2
      Для сферы 2: x 2 — x o = v o t + ½at 2
      x 2 — 1000 = -2t — ½ (-4) t 2
      x 2 = 1000 — 2t — 2t 2
      Теперь найдите время воздействия, установив x 1 = x 2
      10t + t 2 = 1000 — 2t — 2t 2
      0 = -3t 2 -12t +1000
      Используя формулу корней квадратного уравнения: t = 16.4 с
      Найдите скорость (относительно Земли) для каждой сферы во время удара:
      v 1 = = v o + at = 10 + 2 (16,4) = 43 м / с
      v 2 = = V o + at = -2 — 4 (16,4) = -68 м / с
      Их скорость является величиной этих решений.

      10. (умеренно) Эта проблема является продолжением проблемы, которую вы видели в предыдущей презентации об автомобилях на красный свет:
      Машины выстраиваются в линию (с расстоянием 5,0 м между ними) на красный свет.Предположим, что длина каждого вагона составляет 4,6 м. Когда загорится зеленый свет, все автомобили ускоряются со скоростью 1,22 м / с 2 в течение 10,0 секунд, а затем продолжают движение с постоянной скоростью. Если световой индикатор остается зеленым в течение 90,0 секунд, сколько машин доезжают до перекрестка или проезжают через него?
      x 10 = расстояние, на которое каждая машина движется за первые 10 секунд
      x 10 = v o t + ½ при 2 = 0 + ½ (1,22) 10 2 = 61 м
      v 10 = скорость за 10 секунд = v o + при
      v 10 = 0 + (1.22) (10) = 12,2 м / с
      x 80 = расстояние, на которое каждая машина движется за последние 80 секунд
      x 80 = v 10 (80 с) = 976 м
      Общее расстояние, на которое машины перемещаются за 90 с = 61 + 976 = 1037
      Расстояние от передней части одной машины до передней части другой машины = 4,6 + 5 = 9,6 м
      #cars с по с пересечением = 1037 / 9,6 = 108
      Последняя машина подъезжает к краю перекрестка.
      Таким образом, окончательный ответ таков: 109 автомобилей проезжают через пересечение.

      11.(средний) Определите расстояние между двумя стальными сферами (через 1,4 с), сброшенными с башни, если вторая сфера упала через 0,5 секунды после первой. Предположим, что они падают в свободном падении и что шары падают из состояния покоя.
      (используйте y o = 0 и + вверх)
      Сфера 1 через 1,4 с:
      y 1 = y o + v o t + ½gt 2
      y 1 = 0 + 0 — 4,9 (1,4) 2
      y 1 = -9,6 м
      Сфера 2 через 0,9 с:
      y 2 = y o + v o t + ½gt 2
      y 2 = 0 + 0-4.9 (0,9) 2
      y 1 = -4,0 м
      Сферы в это время будут находиться на расстоянии 5,6 м друг от друга.

      12. (жесткий) Если мяч подбрасывается (в условиях свободного падения) с начальной скоростью 2,0 м / с, проводит ли он больше времени в верхних 0,1 м подбрасывания или в нижних 0,1 м подбрасывания?
      (используйте y o = 0 и + вверху)
      Сначала найдите максимальную высоту (где v = 0):
      v 2 = v o 2 + 2gΔy
      0 = 2 2 — 19.6Δy
      Δy = максимальная высота = 0.2 м
      Таким образом, верхнее движение составляет от y = 0,1 м (по пути вверх) до y = 0,1 (по пути вниз):
      Нам нужно найти скорость при y = 0,1:
      v 2 = v o 2 + 2gΔy
      v 2 = 2 2 — 19,6 (0,1)
      v = 1,4 м / с
      (симметрично, скорость при y = 0,1 на пути вниз противоположна)
      Теперь найдите время для верхнего движения:
      v = v o + gt
      -1,4 = 1,4 — 9,8 t
      t = 0,29 с
      Для нижнего движения на пути вверх:
      y — y o = ½ (v + v o ) t
      0.

    Добавить комментарий

    Ваш адрес email не будет опубликован. Обязательные поля помечены *